Approaching Limits with Multiple Variables: How to Choose the Right Method?

  • Thread starter Thread starter loxagos_snake
  • Start date Start date
  • Tags Tags
    Limits Variable
Click For Summary
Calculating limits with multiple variables can be challenging, as demonstrated in the examples provided. For the first limit, the approach using y=mx confirmed that the limit is indeed 0, as the numerator approaches zero while the denominator approaches one. The second limit requires considering different paths for x and y as they approach zero, with the suggestion to set z=0 to simplify the problem. There is no fixed method for these limits; instead, visualizing the function's behavior through various approaches is essential. Understanding the context and experimenting with different techniques will help in determining the appropriate method for each limit.
loxagos_snake
Messages
2
Reaction score
0
OK, our Calc professor gave us a paper with some limits to calculate, but I'm not quite sure about them...Here are some examples:

a)lim (5x^5+3y^5)/(x^2y^2+1)
x->0
y->0

Since the obvious replacement is too suspicious, I thought about looking if it exists by using the y=mx line. Indeed, for different values of m, the limit remained 0, but still I'm not sure if this was the right way.

b)lim (5xy+yz)/(x^2+xz)
x->0
y->0
z->0

I have absolutely no clue about this, except it brings spherical coordinates to my head.

One question I wanted to ask is how I should know which method to use on each limit? Like lines,parabolas,polar coordinates etc.


Anyway, thanks in advance and I apologize if the question format is wrong, it's just my first post!
 
Physics news on Phys.org
For the first one, the obvious is correct. However (x,y) approaches (0,0) the numerator goes to zero and the denominator goes to 1. It's not really an indefinite limit. For the second one, put z=0. Now think about different ways x and y can go to zero. There's not really a fixed 'method' for these problems. You just have to try to visualize how the function behaves by trying different things.
 
Question: A clock's minute hand has length 4 and its hour hand has length 3. What is the distance between the tips at the moment when it is increasing most rapidly?(Putnam Exam Question) Answer: Making assumption that both the hands moves at constant angular velocities, the answer is ## \sqrt{7} .## But don't you think this assumption is somewhat doubtful and wrong?

Similar threads

Replies
2
Views
2K
Replies
35
Views
4K
  • · Replies 8 ·
Replies
8
Views
2K
  • · Replies 1 ·
Replies
1
Views
2K
  • · Replies 12 ·
Replies
12
Views
2K
Replies
5
Views
2K
  • · Replies 5 ·
Replies
5
Views
2K
  • · Replies 4 ·
Replies
4
Views
3K
  • · Replies 4 ·
Replies
4
Views
2K
  • · Replies 1 ·
Replies
1
Views
1K